Quantcast
  • Register
PhysicsOverflow is a next-generation academic platform for physicists and astronomers, including a community peer review system and a postgraduate-level discussion forum analogous to MathOverflow.

Welcome to PhysicsOverflow! PhysicsOverflow is an open platform for community peer review and graduate-level Physics discussion.

Please help promote PhysicsOverflow ads elsewhere if you like it.

News

PO is now at the Physics Department of Bielefeld University!

New printer friendly PO pages!

Migration to Bielefeld University was successful!

Please vote for this year's PhysicsOverflow ads!

Please do help out in categorising submissions. Submit a paper to PhysicsOverflow!

... see more

Tools for paper authors

Submit paper
Claim Paper Authorship

Tools for SE users

Search User
Reclaim SE Account
Request Account Merger
Nativise imported posts
Claim post (deleted users)
Import SE post

Users whose questions have been imported from Physics Stack Exchange, Theoretical Physics Stack Exchange, or any other Stack Exchange site are kindly requested to reclaim their account and not to register as a new user.

Public \(\beta\) tools

Report a bug with a feature
Request a new functionality
404 page design
Send feedback

Attributions

(propose a free ad)

Site Statistics

205 submissions , 163 unreviewed
5,047 questions , 2,200 unanswered
5,345 answers , 22,709 comments
1,470 users with positive rep
816 active unimported users
More ...

  Is there any idea why the electric charges of electron and muon are equal?

+ 6 like - 0 dislike
1714 views

Is there any idea explaining why the electric charges of electron and muon are equal?

Edit:

The total charge of a particle is proportional to the integral of its own electric field flow through the sphere of a big radius surrounding the particle at rest.

The free Dirac equation describes charged fermion. It contains the mass term $m$. If $m$ tends to zero, Dirac equation tends to the pair of Weil equations that describe electrically neutral particles. Does it mean that charge somehow depends on mass? If yes, why do the electron and muon (both described by Dirac equation, but with different mass terms) have the same electric charges?

This post imported from StackExchange Physics at 2015-04-18 05:37 (UTC), posted by SE-user Murod Abdukhakimov
asked Feb 1, 2012 in Theoretical Physics by Murod Abdukhakimov (85 points) [ no revision ]
Is it possible to rephrase the question(v1) in such a way that it does not overlap with this question physics.stackexchange.com/q/4238/2451 ?

This post imported from StackExchange Physics at 2015-04-18 05:37 (UTC), posted by SE-user Qmechanic
Thanks, I didn't see that question. Give me some time to think about rephrasing/closing the question.

This post imported from StackExchange Physics at 2015-04-18 05:37 (UTC), posted by SE-user Murod Abdukhakimov
It's worth googling for "who ordered that" (include the quotation marks as you want to search the phrase).

This post imported from StackExchange Physics at 2015-04-18 05:37 (UTC), posted by SE-user dmckee

2 Answers

+ 7 like - 0 dislike

How do we know that there exists such a particle as the muon?

From observing its decay into an electron plus two other neutral particles, which are an antineutrino electron and a neutrino muon.

In this last sentence there are three conservation laws:

1) conservation of charge ensures that the muon has the same charge as the electron

2) lepton number conservation ensures that the number of particles with muon leptonic number and the number of particles with electron leptonic number are conserved.

These are observations, the accumulation of which together with a large number of other observations allows us to build up the standard model 0f particle physics. The Standard Model encapsulates our observations/data.

The short answer to the question is: because that is what has been observed.

This post imported from StackExchange Physics at 2015-04-18 05:37 (UTC), posted by SE-user anna v
answered Mar 23, 2012 by anna v (2,005 points) [ no revision ]
It is amusing that there exist people who think they are interested in physics and cannot accept an experimental fact, down voting this answer. They need convoluted theoretical arguments which in the end of course end up on the experimental fact. Data trumps theory every single time. Without data theory is science fiction.

This post imported from StackExchange Physics at 2015-04-18 05:37 (UTC), posted by SE-user anna v
Of course you are right, and not my downvote, I think it's an ok answer, but the question seems to be confused about the massless limit of the Dirac equation, not about particle properties.

This post imported from StackExchange Physics at 2015-04-18 05:37 (UTC), posted by SE-user Ron Maimon
+ 4 like - 0 dislike

The confusion is about the Weyl limit--- a massless Weyl fermion can be charged. All the fermions in the standard model are charged Weyl fermions.

The Weyl fermion that can't be charged is the massive Weyl fermion. The reason is that the mass term in the Weyl reduction mixes up the field and its conjugate, so it isn't phase-invariant under multiplying the Weyl field by a complex phase. This type of mass, which is incompatible with charge, is more often called a Majorana mass in the literature, because it is easier to derive as the real part of the Dirac equation in a real basis.

The fact that Weyl fermions can't have mass is important--- it is the reason we see Weyl fermions in nature--- if they could be massive, they would be Planck mass massive. Instead, they are only Higgs-scale massive.

A pair of massive Weyl fermions with the same mass can together be charged, with the charge symmetry rotating one into the other. So a pair of Weyl fermions can also be massive independent of the mass, and the reason is that this is what the Dirac equation is.

The answer to the title question, about the equality of charges, at least from the theoretical standpoint is found here: What is "charge discreteness"? . Anna v. has given the experimental reason.

answered Mar 24, 2012 by Ron Maimon (7,720 points) [ revision history ]
edited Apr 18, 2015 by Ron Maimon

Your answer

Please use answers only to (at least partly) answer questions. To comment, discuss, or ask for clarification, leave a comment instead.
To mask links under text, please type your text, highlight it, and click the "link" button. You can then enter your link URL.
Please consult the FAQ for as to how to format your post.
This is the answer box; if you want to write a comment instead, please use the 'add comment' button.
Live preview (may slow down editor)   Preview
Your name to display (optional):
Privacy: Your email address will only be used for sending these notifications.
Anti-spam verification:
If you are a human please identify the position of the character covered by the symbol $\varnothing$ in the following word:
p$\hbar$ysicsOver$\varnothing$low
Then drag the red bullet below over the corresponding character of our banner. When you drop it there, the bullet changes to green (on slow internet connections after a few seconds).
Please complete the anti-spam verification




user contributions licensed under cc by-sa 3.0 with attribution required

Your rights
...